LSAT and Law School Admissions Forum

Get expert LSAT preparation and law school admissions advice from PowerScore Test Preparation.

 Administrator
PowerScore Staff
  • PowerScore Staff
  • Posts: 8916
  • Joined: Feb 02, 2011
|
#40964
Complete Question Explanation

Weaken, CE. The correct answer choice is (E)


The politician uses an analogy to support his conclusion: because the privatization of the telecommunications industry benefited consumers by allowing competition among a variety of telephone companies, the politician concludes that same benefit will result from the privatization of national parks. For the benefits of privatization to be analogous in this respect, it is necessary that national parks and the telecommunications industry be fundamentally similar. However, if they were not, then the author would be guilty of a False Analogy.

The argument is structured as follows:
PT62 - LR2 #14 diagram 1.png
Given that the question stem asks us to weaken the argument, you should attack the answers by looking for a statement that, if true, would show that national parks and the telecommunications industry are different in a way that undermines the credibility of the author’s conclusion. More specifically, you should notice that the premise regarding the telecommunications industry relies on a crucial link between privatization and consumer benefit: the increased competition among companies, which forced down prices and improved services. Since there is no evidence that privatizing national parks would result in the same increased competition, this may prove detrimental to the strength of the analogy used to support the conclusion.

Remember—in an argument by analogy, the author uses the presumption of similarity between two things to argue that they probably have additional traits in common. Analogies are often used to clarify the relationship between the items or reveal a fundamental truth about one of the items. When used properly, an analogy can be a powerful tool of argumentation. However, if the author treats as similar two cases that are different in a critical respect, then the analogy is false and the conclusion is questionable. If you see an analogy in the stimulus followed by a Weaken question stem, focus on undermining the strength of the analogy.

Answer choice (A): Since the conclusion is confined to the probable benefit of privatization to park visitors, it is irrelevant whether such course of action is politically expedient. This is a classic Shell Game answer, used to attack a conclusion that is similar to, but slightly different from, the one presented in the stimulus. Had the author concluded, “it is feasible to privatize national parks,” answer choice (A) would have been more attractive.

Savvy test takers would disregard answer choice (A) simply because it does not address the dubious analogy used in the stimulus.

Answer choice (B): Although this answer choice presents a downside to the privatization of the telecommunications industry, such privatization clearly affected only its employees and the industry in general, not its customers who ultimately benefited from the lower prices. Therefore, even if the privatization of national parks led to a similar increase in unemployment of park rangers and other service personnel, this would not necessarily affect the prices or the quality of services provided to park visitors.

Answer choice (C): Hopefully, you were able to eliminate this answer choice quickly. Even if park visitors were unaware of any proposals to privatize the management of parks, this does not mean that they won’t benefit from such proposals.

Answer choice (D): At first glance, this may seem like an attractive answer, given the attempted attack on the analogy between national parks and the telecommunications industry. However, this attack is pointless because it focuses on an irrelevant distinction between the two.

Even if privatizing the national parks would benefit a much smaller number of consumers to a much smaller extent than did the privatization of the telecommunications industry, it is still possible that the privatization of parks benefits most (if not all) park visitors. This is because it is entirely plausible that the number of park visitors is much smaller than the number of, say, cell phone users, which would explain why a lot fewer people would benefit from privatized national parks. Furthermore, just because the extent of the benefit to park visitors is smaller when compared to the benefit to cell phone users does not mean that the benefit is not real or valuable. As long as the benefits of privatization outweigh the costs to park visitors, the proposal to privatize the national parks has merit.

Answer choice (E): This is the correct answer choice. If privatizing the national parks produces much less competition between different companies than did privatizing the telecommunications industry, then it is unreasonable to expect that park visitors would benefit from the proposal to privatize the parks. After all, the reason why privatization of the telecommunications industry benefited consumers is that it allowed competition among a variety of telephone companies to improve service and force down prices. If the intermediate effect of increased competition does not occur as a result of privatizing the national parks, then the causal chain is interrupted and the analogy between the two falls apart.
You do not have the required permissions to view the files attached to this post.
 moshei24
  • Posts: 465
  • Joined: Mar 20, 2012
|
#5673
I look at this question and I see that the assumption is just like privatizing telephones benefited their consumers due to competition, it privatization of national parks will benefit their consumer due to competition.

(E) seems to be right because if there's much less competition in national parks, maybe it will no longer have the effect that it had with telephones. But who cares if it has the same effect? Even if the effect is much less, it will still benefit them. Or is the real reason this answer is correct is because much less competition might mean that the benefits that telephones had wouldn't occur at all with national parks? That the level of competition that telephones had caused all the benefits, but a much lower level of competition won't have any benefits? So it casts doubt on the assumption that the competition that would result from the privatization would be enough to cause any benefit?

Do I understand that properly? If I'm off, please explain to me where, even if I'm only slightly off.

Thanks!
 Steve Stein
PowerScore Staff
  • PowerScore Staff
  • Posts: 1153
  • Joined: Apr 11, 2011
|
#5683
Hi Moshe,

The author of that stimulus proposes privatization of national parks, comparing the move to the privatization of telecommunications, which increased competition, leading to improved services and decreased prices.

The stimulus is followed by a Weaken question, and the correct answer choice provides that park privatization would produce much less competition. This weakens the analogy to telecommunication privatization, and thus weakens the author's argument.

I hope that clears it up--let me know. Thanks!

~Steve
 moshei24
  • Posts: 465
  • Joined: Mar 20, 2012
|
#5685
Oh, okay. Weakening the analogy weakens the argument. Got it.
 netherlands
  • Posts: 136
  • Joined: Apr 17, 2013
|
#9920
Hi there PS,

I've seen questions like this before where the author makes a proposal based off of an example that had a specific outcome. In order to weaken something like this - is E the correct choice because it attacks the exact benefit that the example is based on?

I can see how A, B and C are irrelevant. But D and E both seemed to be on the same level to me-both basically say that making this move to privatization would be less effective or have less of an impact in the National Parks industry than it did in the telecom industry - Is E the better choices bc it explicitly states that the exact benefit the stimulus based its suggestion off of would be less likely? Because to me both of them are saying that the move could give benefits but that either way it'd be to a much lesser extent than with the telecom industry.
 Jason Schultz
PowerScore Staff
  • PowerScore Staff
  • Posts: 49
  • Joined: Jun 13, 2013
|
#9930
Hi netherlands,

The stimulus uses causal reasoning to conclude that a specific course of action (privatization) will have a benefit for consumers/park visitors. However, the middle term in that argument is "competition." Put more simple, the Politicians argument is that Privatization causes competition which causes lower prices/better service.

Answer choice D notes that that benefit will be smaller than it would be for telecom, but it does confirm that a benefit will ultimately result. Since the politician only concludes that privatization will "probably" benefit park visitors, this actually strengthens his conclusion. Admittedly by a small margin, but that's all that's required.

However, Answer choice E directly attacks his causal chain. If privatization doesn't cause competition, then it may not result in any benefit whatsoever. That directly attacks the conclusion.
 Lina
  • Posts: 22
  • Joined: Aug 05, 2013
|
#10181
December 2010, LR 1

Q#14
I would greatly appreciate tips on how to avoid the trap I fell into with choice D. I looked for something that would weaken the benefits of having a privatized national park. I did not look for something that would challenge the competition factor.
 David Boyle
PowerScore Staff
  • PowerScore Staff
  • Posts: 836
  • Joined: Jun 07, 2013
|
#10202
Lina wrote:December 2010, LR 1

Q#14
I would greatly appreciate tips on how to avoid the trap I fell into with choice D. I looked for something that would weaken the benefits of having a privatized national park. I did not look for something that would challenge the competition factor.
Dear Lina:

With answer choice D, consumers are still benefited. Answer E goes a step back to see if the mechanism that even produces the benefit at all, works. In the stimulus, the mechanism is competition. E attacks the very idea that competition will work well to produce even any benefit at all.
So, it's sort of a numbers thing in a way. The stimulus didn't say there'd be **as much as or more benefit** for privatizing the parks, just that there'd be benefit ("probably"). Therefore, attacking the amount of benefit might not help.
Reading the question carefully and seeing what's needed will be helpful. And it may not be the thing that you suspect at first. It may be something back on the temporal chain, e.g., here, what produces the benefit, not the benefit itself, which comes later.
Hope that helps,

David
 tetrisyong
  • Posts: 1
  • Joined: Mar 16, 2019
|
#63437
I understand that E attacks the causal chain of the conclusion: generated competition will be beneficial. However, I had problem with the ‘much less’ part. I thought that much less doesn’t equal 0. So even if the competition will be much less, privatization will still generate some amount of competition - thus still be beneficial, which I considered is not strong enough to be the answer for weaken. Can someone help me understand this part?
 Robert Carroll
PowerScore Staff
  • PowerScore Staff
  • Posts: 1787
  • Joined: Dec 06, 2013
|
#63470
tetris,

First, "much less competition" is compatible with 0 competition. The phrase doesn't require that there be no competition, but the phrase "much less" just means "significantly lower." If national park privatization resulted in no competition whatsoever, that certainly would count as "much less" competition. So, I think it's worthwhile to clarify that "much less" ALLOWS for 0, but doesn't require it. The case is analogous to that of the phrase "not all". "Not all" means "anything less than all." 0 is less than all. So "not all" COVERS the number 0, but it also covers any number lower than "all". So it's correct to say "'not all' doesn't equal 0", but be careful to distinguish that from the incorrect statement "'not all' excludes the possibility of 0." That just isn't true.

Here, "much less" allows 0, but doesn't require it. 0 is one of many degrees of competition allowed by the phrase.

Second, because "much less" means some degree less, but not necessarily the absence of competition, you're correct that this means there COULD be some competition in the national parks. Thus, answer choice (E) does not prove the author wrong. But this is a Weaken question. The point of the answer isn't necessarily to prove the author wrong, but to make it less likely, by whatever degree, that the author is correct. So the objection that answer choice (E) doesn't decisively refute the author is misguided. You should ask instead "Does this answer make the author less likely to be correct?" And answer choice (E) certainly does make the author less likely to be correct. This shows that the answer weakens the argument.

It is good always to keep in mind that the answer to a Weaken question does not have to provide a full refutation to the argument. In fact, the typical case for a correct answer to a Weaken question involves undermining the author's argument to some extent, not fully refuting the argument.

Robert Carroll

Get the most out of your LSAT Prep Plus subscription.

Analyze and track your performance with our Testing and Analytics Package.